Difference between revisions of "1987 AHSME Problems/Problem 30"

(Created page with "==Problem== In the figure, <math>\triangle ABC</math> has <math>\angle A =45^{\circ}</math> and <math>\angle B =30^{\circ}</math>. A line <math>DE</math>, with <math>D</math> on...")
 
m (Problem)
Line 3: Line 3:
 
In the figure, <math>\triangle ABC</math> has <math>\angle A =45^{\circ}</math> and <math>\angle B =30^{\circ}</math>. A line <math>DE</math>, with <math>D</math> on <math>AB</math>  
 
In the figure, <math>\triangle ABC</math> has <math>\angle A =45^{\circ}</math> and <math>\angle B =30^{\circ}</math>. A line <math>DE</math>, with <math>D</math> on <math>AB</math>  
 
and <math>\angle ADE =60^{\circ}</math>, divides <math>\triangle ABC</math> into two pieces of equal area.  
 
and <math>\angle ADE =60^{\circ}</math>, divides <math>\triangle ABC</math> into two pieces of equal area.  
(Note: the figure may not be accurate; perhaps <math>E</math> is on <math>CB</math> instead of <math>AC</math>.)
+
(Note: the figure may not be accurate; perhaps <math>E</math> is on <math>CB</math> instead of <math>AC.) </math>
 
The ratio <math>\frac{AD}{AB}</math> is
 
The ratio <math>\frac{AD}{AB}</math> is
  
Line 25: Line 25:
 
\textbf{(D)}\ \frac{1}{\sqrt[3]{6}}\qquad
 
\textbf{(D)}\ \frac{1}{\sqrt[3]{6}}\qquad
 
\textbf{(E)}\ \frac{1}{\sqrt[4]{12}}  </math>
 
\textbf{(E)}\ \frac{1}{\sqrt[4]{12}}  </math>
 
  
 
== See also ==
 
== See also ==

Revision as of 08:56, 23 October 2014

Problem

In the figure, $\triangle ABC$ has $\angle A =45^{\circ}$ and $\angle B =30^{\circ}$. A line $DE$, with $D$ on $AB$ and $\angle ADE =60^{\circ}$, divides $\triangle ABC$ into two pieces of equal area. (Note: the figure may not be accurate; perhaps $E$ is on $CB$ instead of $AC.)$ The ratio $\frac{AD}{AB}$ is

[asy] size((220)); draw((0,0)--(20,0)--(7,6)--cycle); draw((6,6)--(10,-1)); label("A", (0,0), W); label("B", (20,0), E); label("C", (7,6), NE); label("D", (9.5,-1), W); label("E", (5.9, 6.1), SW); label("$45^{\circ}$", (2.5,.5)); label("$60^{\circ}$", (7.8,.5)); label("$30^{\circ}$", (16.5,.5)); [/asy]

$\textbf{(A)}\ \frac{1}{\sqrt{2}} \qquad \textbf{(B)}\ \frac{2}{2+\sqrt{2}} \qquad \textbf{(C)}\ \frac{1}{\sqrt{3}} \qquad \textbf{(D)}\ \frac{1}{\sqrt[3]{6}}\qquad \textbf{(E)}\ \frac{1}{\sqrt[4]{12}}$

See also

1987 AHSME (ProblemsAnswer KeyResources)
Preceded by
Problem 29
Followed by
Last Question
1 2 3 4 5 6 7 8 9 10 11 12 13 14 15 16 17 18 19 20 21 22 23 24 25 26 27 28 29 30
All AHSME Problems and Solutions

The problems on this page are copyrighted by the Mathematical Association of America's American Mathematics Competitions. AMC logo.png